Non-zero continuous function $f(x)$ such that $int_0^1x^k (1-x)^{n-k} f(x)dx=0$ for every $k=0,1,…,n$ and...












1












$begingroup$


Does there exists a non-zero continuous function $f$ such that $displaystyle int_0^1 x^k (1-x)^{n-k} f(x)dx=0$ for every $k=0,1,2,...,n$ where $n$ is a non-negative integer.



EDIT: The problem is related to Weierstrass approximation using Bernstein approximation of certain function. As you can see the term $x^k (1-x)^{n-k}$ comes from Bernstein polynomials. I want my integral to vanish for non-zero $f$.



Also, the result should hold for every $n$ and for every $k$ running from $0$ to $n$. For example, for $n=2$, we shall have three integrals corresponding to $k=0,1,2$. In this way, it should hold for any $n$.










share|cite|improve this question











$endgroup$












  • $begingroup$
    This question is missing context or other details: Please improve the question by providing additional context, which ideally includes your thoughts on the problem and any attempts you have made to solve it. This information helps others identify where you have difficulties and helps them write answers appropriate to your experience level.
    $endgroup$
    – Rory Daulton
    Jan 20 at 0:45










  • $begingroup$
    Do you mean everywhere nonzero or not identically 0?
    $endgroup$
    – Van Latimer
    Jan 20 at 0:48






  • 2




    $begingroup$
    The set $$ V_n = left{ f in L^2([0,1]) : int_{0}^{1} x^k(1-x)^{n-k}f(x) , mathrm{d}x = 0 text{ for every } k = 0, cdots, n right} $$ is the orthogonal complement of the set of all polynomials of degree $leq n$, and in particular, is not only non-empty but in fact an infinite-dimensional subspace of $L^{2}([0,1])$. One can find an explicit orthogonal basis of $V_n$, for instance, using the shifted Legendre polynomials, which are of course continuous.
    $endgroup$
    – Sangchul Lee
    Jan 20 at 1:10








  • 1




    $begingroup$
    Ah, I thought that you are asking examples of $f$ for each given $n$. If the condition should hold for all $n$, then $f$ must be identically zero, by the standard approximation technique, i.e., choose a sequence of polynomials $p_n$ that approximate $f$ in $C([0,1])$ to see that $$ int_{0}^{1} f(x)^2 , mathrm{d}x = lim_{ntoinfty}int_{0}^{1} f(x)p_n(x) , mathrm{d}x stackrel{text{(by assumption)}}= 0. $$ This is enough to conclude $f equiv 0$. The conclusion continues to hold (modulo measure-zero modification) for larger class of functions.
    $endgroup$
    – Sangchul Lee
    Jan 20 at 2:50








  • 1




    $begingroup$
    If $f$ is $L^1$, then one can show that its Fourier transform $hat{f}(xi) = int_{0}^{1}f(x)e^{-2pi ixi x} , mathrm{d}x$ is identically zero, which is enough to show that $f$ is zero a.e.
    $endgroup$
    – Sangchul Lee
    Jan 20 at 5:06
















1












$begingroup$


Does there exists a non-zero continuous function $f$ such that $displaystyle int_0^1 x^k (1-x)^{n-k} f(x)dx=0$ for every $k=0,1,2,...,n$ where $n$ is a non-negative integer.



EDIT: The problem is related to Weierstrass approximation using Bernstein approximation of certain function. As you can see the term $x^k (1-x)^{n-k}$ comes from Bernstein polynomials. I want my integral to vanish for non-zero $f$.



Also, the result should hold for every $n$ and for every $k$ running from $0$ to $n$. For example, for $n=2$, we shall have three integrals corresponding to $k=0,1,2$. In this way, it should hold for any $n$.










share|cite|improve this question











$endgroup$












  • $begingroup$
    This question is missing context or other details: Please improve the question by providing additional context, which ideally includes your thoughts on the problem and any attempts you have made to solve it. This information helps others identify where you have difficulties and helps them write answers appropriate to your experience level.
    $endgroup$
    – Rory Daulton
    Jan 20 at 0:45










  • $begingroup$
    Do you mean everywhere nonzero or not identically 0?
    $endgroup$
    – Van Latimer
    Jan 20 at 0:48






  • 2




    $begingroup$
    The set $$ V_n = left{ f in L^2([0,1]) : int_{0}^{1} x^k(1-x)^{n-k}f(x) , mathrm{d}x = 0 text{ for every } k = 0, cdots, n right} $$ is the orthogonal complement of the set of all polynomials of degree $leq n$, and in particular, is not only non-empty but in fact an infinite-dimensional subspace of $L^{2}([0,1])$. One can find an explicit orthogonal basis of $V_n$, for instance, using the shifted Legendre polynomials, which are of course continuous.
    $endgroup$
    – Sangchul Lee
    Jan 20 at 1:10








  • 1




    $begingroup$
    Ah, I thought that you are asking examples of $f$ for each given $n$. If the condition should hold for all $n$, then $f$ must be identically zero, by the standard approximation technique, i.e., choose a sequence of polynomials $p_n$ that approximate $f$ in $C([0,1])$ to see that $$ int_{0}^{1} f(x)^2 , mathrm{d}x = lim_{ntoinfty}int_{0}^{1} f(x)p_n(x) , mathrm{d}x stackrel{text{(by assumption)}}= 0. $$ This is enough to conclude $f equiv 0$. The conclusion continues to hold (modulo measure-zero modification) for larger class of functions.
    $endgroup$
    – Sangchul Lee
    Jan 20 at 2:50








  • 1




    $begingroup$
    If $f$ is $L^1$, then one can show that its Fourier transform $hat{f}(xi) = int_{0}^{1}f(x)e^{-2pi ixi x} , mathrm{d}x$ is identically zero, which is enough to show that $f$ is zero a.e.
    $endgroup$
    – Sangchul Lee
    Jan 20 at 5:06














1












1








1


1



$begingroup$


Does there exists a non-zero continuous function $f$ such that $displaystyle int_0^1 x^k (1-x)^{n-k} f(x)dx=0$ for every $k=0,1,2,...,n$ where $n$ is a non-negative integer.



EDIT: The problem is related to Weierstrass approximation using Bernstein approximation of certain function. As you can see the term $x^k (1-x)^{n-k}$ comes from Bernstein polynomials. I want my integral to vanish for non-zero $f$.



Also, the result should hold for every $n$ and for every $k$ running from $0$ to $n$. For example, for $n=2$, we shall have three integrals corresponding to $k=0,1,2$. In this way, it should hold for any $n$.










share|cite|improve this question











$endgroup$




Does there exists a non-zero continuous function $f$ such that $displaystyle int_0^1 x^k (1-x)^{n-k} f(x)dx=0$ for every $k=0,1,2,...,n$ where $n$ is a non-negative integer.



EDIT: The problem is related to Weierstrass approximation using Bernstein approximation of certain function. As you can see the term $x^k (1-x)^{n-k}$ comes from Bernstein polynomials. I want my integral to vanish for non-zero $f$.



Also, the result should hold for every $n$ and for every $k$ running from $0$ to $n$. For example, for $n=2$, we shall have three integrals corresponding to $k=0,1,2$. In this way, it should hold for any $n$.







real-analysis calculus integration analysis lebesgue-integral






share|cite|improve this question















share|cite|improve this question













share|cite|improve this question




share|cite|improve this question








edited Jan 20 at 1:03







ersh

















asked Jan 20 at 0:36









ershersh

408113




408113












  • $begingroup$
    This question is missing context or other details: Please improve the question by providing additional context, which ideally includes your thoughts on the problem and any attempts you have made to solve it. This information helps others identify where you have difficulties and helps them write answers appropriate to your experience level.
    $endgroup$
    – Rory Daulton
    Jan 20 at 0:45










  • $begingroup$
    Do you mean everywhere nonzero or not identically 0?
    $endgroup$
    – Van Latimer
    Jan 20 at 0:48






  • 2




    $begingroup$
    The set $$ V_n = left{ f in L^2([0,1]) : int_{0}^{1} x^k(1-x)^{n-k}f(x) , mathrm{d}x = 0 text{ for every } k = 0, cdots, n right} $$ is the orthogonal complement of the set of all polynomials of degree $leq n$, and in particular, is not only non-empty but in fact an infinite-dimensional subspace of $L^{2}([0,1])$. One can find an explicit orthogonal basis of $V_n$, for instance, using the shifted Legendre polynomials, which are of course continuous.
    $endgroup$
    – Sangchul Lee
    Jan 20 at 1:10








  • 1




    $begingroup$
    Ah, I thought that you are asking examples of $f$ for each given $n$. If the condition should hold for all $n$, then $f$ must be identically zero, by the standard approximation technique, i.e., choose a sequence of polynomials $p_n$ that approximate $f$ in $C([0,1])$ to see that $$ int_{0}^{1} f(x)^2 , mathrm{d}x = lim_{ntoinfty}int_{0}^{1} f(x)p_n(x) , mathrm{d}x stackrel{text{(by assumption)}}= 0. $$ This is enough to conclude $f equiv 0$. The conclusion continues to hold (modulo measure-zero modification) for larger class of functions.
    $endgroup$
    – Sangchul Lee
    Jan 20 at 2:50








  • 1




    $begingroup$
    If $f$ is $L^1$, then one can show that its Fourier transform $hat{f}(xi) = int_{0}^{1}f(x)e^{-2pi ixi x} , mathrm{d}x$ is identically zero, which is enough to show that $f$ is zero a.e.
    $endgroup$
    – Sangchul Lee
    Jan 20 at 5:06


















  • $begingroup$
    This question is missing context or other details: Please improve the question by providing additional context, which ideally includes your thoughts on the problem and any attempts you have made to solve it. This information helps others identify where you have difficulties and helps them write answers appropriate to your experience level.
    $endgroup$
    – Rory Daulton
    Jan 20 at 0:45










  • $begingroup$
    Do you mean everywhere nonzero or not identically 0?
    $endgroup$
    – Van Latimer
    Jan 20 at 0:48






  • 2




    $begingroup$
    The set $$ V_n = left{ f in L^2([0,1]) : int_{0}^{1} x^k(1-x)^{n-k}f(x) , mathrm{d}x = 0 text{ for every } k = 0, cdots, n right} $$ is the orthogonal complement of the set of all polynomials of degree $leq n$, and in particular, is not only non-empty but in fact an infinite-dimensional subspace of $L^{2}([0,1])$. One can find an explicit orthogonal basis of $V_n$, for instance, using the shifted Legendre polynomials, which are of course continuous.
    $endgroup$
    – Sangchul Lee
    Jan 20 at 1:10








  • 1




    $begingroup$
    Ah, I thought that you are asking examples of $f$ for each given $n$. If the condition should hold for all $n$, then $f$ must be identically zero, by the standard approximation technique, i.e., choose a sequence of polynomials $p_n$ that approximate $f$ in $C([0,1])$ to see that $$ int_{0}^{1} f(x)^2 , mathrm{d}x = lim_{ntoinfty}int_{0}^{1} f(x)p_n(x) , mathrm{d}x stackrel{text{(by assumption)}}= 0. $$ This is enough to conclude $f equiv 0$. The conclusion continues to hold (modulo measure-zero modification) for larger class of functions.
    $endgroup$
    – Sangchul Lee
    Jan 20 at 2:50








  • 1




    $begingroup$
    If $f$ is $L^1$, then one can show that its Fourier transform $hat{f}(xi) = int_{0}^{1}f(x)e^{-2pi ixi x} , mathrm{d}x$ is identically zero, which is enough to show that $f$ is zero a.e.
    $endgroup$
    – Sangchul Lee
    Jan 20 at 5:06
















$begingroup$
This question is missing context or other details: Please improve the question by providing additional context, which ideally includes your thoughts on the problem and any attempts you have made to solve it. This information helps others identify where you have difficulties and helps them write answers appropriate to your experience level.
$endgroup$
– Rory Daulton
Jan 20 at 0:45




$begingroup$
This question is missing context or other details: Please improve the question by providing additional context, which ideally includes your thoughts on the problem and any attempts you have made to solve it. This information helps others identify where you have difficulties and helps them write answers appropriate to your experience level.
$endgroup$
– Rory Daulton
Jan 20 at 0:45












$begingroup$
Do you mean everywhere nonzero or not identically 0?
$endgroup$
– Van Latimer
Jan 20 at 0:48




$begingroup$
Do you mean everywhere nonzero or not identically 0?
$endgroup$
– Van Latimer
Jan 20 at 0:48




2




2




$begingroup$
The set $$ V_n = left{ f in L^2([0,1]) : int_{0}^{1} x^k(1-x)^{n-k}f(x) , mathrm{d}x = 0 text{ for every } k = 0, cdots, n right} $$ is the orthogonal complement of the set of all polynomials of degree $leq n$, and in particular, is not only non-empty but in fact an infinite-dimensional subspace of $L^{2}([0,1])$. One can find an explicit orthogonal basis of $V_n$, for instance, using the shifted Legendre polynomials, which are of course continuous.
$endgroup$
– Sangchul Lee
Jan 20 at 1:10






$begingroup$
The set $$ V_n = left{ f in L^2([0,1]) : int_{0}^{1} x^k(1-x)^{n-k}f(x) , mathrm{d}x = 0 text{ for every } k = 0, cdots, n right} $$ is the orthogonal complement of the set of all polynomials of degree $leq n$, and in particular, is not only non-empty but in fact an infinite-dimensional subspace of $L^{2}([0,1])$. One can find an explicit orthogonal basis of $V_n$, for instance, using the shifted Legendre polynomials, which are of course continuous.
$endgroup$
– Sangchul Lee
Jan 20 at 1:10






1




1




$begingroup$
Ah, I thought that you are asking examples of $f$ for each given $n$. If the condition should hold for all $n$, then $f$ must be identically zero, by the standard approximation technique, i.e., choose a sequence of polynomials $p_n$ that approximate $f$ in $C([0,1])$ to see that $$ int_{0}^{1} f(x)^2 , mathrm{d}x = lim_{ntoinfty}int_{0}^{1} f(x)p_n(x) , mathrm{d}x stackrel{text{(by assumption)}}= 0. $$ This is enough to conclude $f equiv 0$. The conclusion continues to hold (modulo measure-zero modification) for larger class of functions.
$endgroup$
– Sangchul Lee
Jan 20 at 2:50






$begingroup$
Ah, I thought that you are asking examples of $f$ for each given $n$. If the condition should hold for all $n$, then $f$ must be identically zero, by the standard approximation technique, i.e., choose a sequence of polynomials $p_n$ that approximate $f$ in $C([0,1])$ to see that $$ int_{0}^{1} f(x)^2 , mathrm{d}x = lim_{ntoinfty}int_{0}^{1} f(x)p_n(x) , mathrm{d}x stackrel{text{(by assumption)}}= 0. $$ This is enough to conclude $f equiv 0$. The conclusion continues to hold (modulo measure-zero modification) for larger class of functions.
$endgroup$
– Sangchul Lee
Jan 20 at 2:50






1




1




$begingroup$
If $f$ is $L^1$, then one can show that its Fourier transform $hat{f}(xi) = int_{0}^{1}f(x)e^{-2pi ixi x} , mathrm{d}x$ is identically zero, which is enough to show that $f$ is zero a.e.
$endgroup$
– Sangchul Lee
Jan 20 at 5:06




$begingroup$
If $f$ is $L^1$, then one can show that its Fourier transform $hat{f}(xi) = int_{0}^{1}f(x)e^{-2pi ixi x} , mathrm{d}x$ is identically zero, which is enough to show that $f$ is zero a.e.
$endgroup$
– Sangchul Lee
Jan 20 at 5:06










1 Answer
1






active

oldest

votes


















3












$begingroup$

Here’s the sketch: Since we require this for every $k$ and $n$, it amounts to saying that the integral of f against any $x^k (1-x)^j$ is zero. These are closed under multiplication, so their span is an algebra. By Stone-Weierstrass, find linear combinations $l_n$ of them approaching $f$ uniformly. By the assumption in the problem, $int l_n f$ is 0 for all $n$, so taking a limit we conclude $f^2$ integrates to 0. It however is positive, so it is identically 0. Therefore $f$ is identically 0.






share|cite|improve this answer









$endgroup$













  • $begingroup$
    @ Van Latimer, It really seems such $f$ must be zero. I am reading your proof.
    $endgroup$
    – ersh
    Jan 20 at 2:41












  • $begingroup$
    Right, sorry I didn’t state that explicitly till the end.
    $endgroup$
    – Van Latimer
    Jan 20 at 3:20











Your Answer





StackExchange.ifUsing("editor", function () {
return StackExchange.using("mathjaxEditing", function () {
StackExchange.MarkdownEditor.creationCallbacks.add(function (editor, postfix) {
StackExchange.mathjaxEditing.prepareWmdForMathJax(editor, postfix, [["$", "$"], ["\\(","\\)"]]);
});
});
}, "mathjax-editing");

StackExchange.ready(function() {
var channelOptions = {
tags: "".split(" "),
id: "69"
};
initTagRenderer("".split(" "), "".split(" "), channelOptions);

StackExchange.using("externalEditor", function() {
// Have to fire editor after snippets, if snippets enabled
if (StackExchange.settings.snippets.snippetsEnabled) {
StackExchange.using("snippets", function() {
createEditor();
});
}
else {
createEditor();
}
});

function createEditor() {
StackExchange.prepareEditor({
heartbeatType: 'answer',
autoActivateHeartbeat: false,
convertImagesToLinks: true,
noModals: true,
showLowRepImageUploadWarning: true,
reputationToPostImages: 10,
bindNavPrevention: true,
postfix: "",
imageUploader: {
brandingHtml: "Powered by u003ca class="icon-imgur-white" href="https://imgur.com/"u003eu003c/au003e",
contentPolicyHtml: "User contributions licensed under u003ca href="https://creativecommons.org/licenses/by-sa/3.0/"u003ecc by-sa 3.0 with attribution requiredu003c/au003e u003ca href="https://stackoverflow.com/legal/content-policy"u003e(content policy)u003c/au003e",
allowUrls: true
},
noCode: true, onDemand: true,
discardSelector: ".discard-answer"
,immediatelyShowMarkdownHelp:true
});


}
});














draft saved

draft discarded


















StackExchange.ready(
function () {
StackExchange.openid.initPostLogin('.new-post-login', 'https%3a%2f%2fmath.stackexchange.com%2fquestions%2f3080018%2fnon-zero-continuous-function-fx-such-that-int-01xk-1-xn-k-fxdx-0%23new-answer', 'question_page');
}
);

Post as a guest















Required, but never shown

























1 Answer
1






active

oldest

votes








1 Answer
1






active

oldest

votes









active

oldest

votes






active

oldest

votes









3












$begingroup$

Here’s the sketch: Since we require this for every $k$ and $n$, it amounts to saying that the integral of f against any $x^k (1-x)^j$ is zero. These are closed under multiplication, so their span is an algebra. By Stone-Weierstrass, find linear combinations $l_n$ of them approaching $f$ uniformly. By the assumption in the problem, $int l_n f$ is 0 for all $n$, so taking a limit we conclude $f^2$ integrates to 0. It however is positive, so it is identically 0. Therefore $f$ is identically 0.






share|cite|improve this answer









$endgroup$













  • $begingroup$
    @ Van Latimer, It really seems such $f$ must be zero. I am reading your proof.
    $endgroup$
    – ersh
    Jan 20 at 2:41












  • $begingroup$
    Right, sorry I didn’t state that explicitly till the end.
    $endgroup$
    – Van Latimer
    Jan 20 at 3:20
















3












$begingroup$

Here’s the sketch: Since we require this for every $k$ and $n$, it amounts to saying that the integral of f against any $x^k (1-x)^j$ is zero. These are closed under multiplication, so their span is an algebra. By Stone-Weierstrass, find linear combinations $l_n$ of them approaching $f$ uniformly. By the assumption in the problem, $int l_n f$ is 0 for all $n$, so taking a limit we conclude $f^2$ integrates to 0. It however is positive, so it is identically 0. Therefore $f$ is identically 0.






share|cite|improve this answer









$endgroup$













  • $begingroup$
    @ Van Latimer, It really seems such $f$ must be zero. I am reading your proof.
    $endgroup$
    – ersh
    Jan 20 at 2:41












  • $begingroup$
    Right, sorry I didn’t state that explicitly till the end.
    $endgroup$
    – Van Latimer
    Jan 20 at 3:20














3












3








3





$begingroup$

Here’s the sketch: Since we require this for every $k$ and $n$, it amounts to saying that the integral of f against any $x^k (1-x)^j$ is zero. These are closed under multiplication, so their span is an algebra. By Stone-Weierstrass, find linear combinations $l_n$ of them approaching $f$ uniformly. By the assumption in the problem, $int l_n f$ is 0 for all $n$, so taking a limit we conclude $f^2$ integrates to 0. It however is positive, so it is identically 0. Therefore $f$ is identically 0.






share|cite|improve this answer









$endgroup$



Here’s the sketch: Since we require this for every $k$ and $n$, it amounts to saying that the integral of f against any $x^k (1-x)^j$ is zero. These are closed under multiplication, so their span is an algebra. By Stone-Weierstrass, find linear combinations $l_n$ of them approaching $f$ uniformly. By the assumption in the problem, $int l_n f$ is 0 for all $n$, so taking a limit we conclude $f^2$ integrates to 0. It however is positive, so it is identically 0. Therefore $f$ is identically 0.







share|cite|improve this answer












share|cite|improve this answer



share|cite|improve this answer










answered Jan 20 at 2:13









Van LatimerVan Latimer

321110




321110












  • $begingroup$
    @ Van Latimer, It really seems such $f$ must be zero. I am reading your proof.
    $endgroup$
    – ersh
    Jan 20 at 2:41












  • $begingroup$
    Right, sorry I didn’t state that explicitly till the end.
    $endgroup$
    – Van Latimer
    Jan 20 at 3:20


















  • $begingroup$
    @ Van Latimer, It really seems such $f$ must be zero. I am reading your proof.
    $endgroup$
    – ersh
    Jan 20 at 2:41












  • $begingroup$
    Right, sorry I didn’t state that explicitly till the end.
    $endgroup$
    – Van Latimer
    Jan 20 at 3:20
















$begingroup$
@ Van Latimer, It really seems such $f$ must be zero. I am reading your proof.
$endgroup$
– ersh
Jan 20 at 2:41






$begingroup$
@ Van Latimer, It really seems such $f$ must be zero. I am reading your proof.
$endgroup$
– ersh
Jan 20 at 2:41














$begingroup$
Right, sorry I didn’t state that explicitly till the end.
$endgroup$
– Van Latimer
Jan 20 at 3:20




$begingroup$
Right, sorry I didn’t state that explicitly till the end.
$endgroup$
– Van Latimer
Jan 20 at 3:20


















draft saved

draft discarded




















































Thanks for contributing an answer to Mathematics Stack Exchange!


  • Please be sure to answer the question. Provide details and share your research!

But avoid



  • Asking for help, clarification, or responding to other answers.

  • Making statements based on opinion; back them up with references or personal experience.


Use MathJax to format equations. MathJax reference.


To learn more, see our tips on writing great answers.




draft saved


draft discarded














StackExchange.ready(
function () {
StackExchange.openid.initPostLogin('.new-post-login', 'https%3a%2f%2fmath.stackexchange.com%2fquestions%2f3080018%2fnon-zero-continuous-function-fx-such-that-int-01xk-1-xn-k-fxdx-0%23new-answer', 'question_page');
}
);

Post as a guest















Required, but never shown





















































Required, but never shown














Required, but never shown












Required, but never shown







Required, but never shown

































Required, but never shown














Required, but never shown












Required, but never shown







Required, but never shown







Popular posts from this blog

Mario Kart Wii

What does “Dominus providebit” mean?

Antonio Litta Visconti Arese